Last visit was: 26 Apr 2024, 00:57 It is currently 26 Apr 2024, 00:57

Close
GMAT Club Daily Prep
Thank you for using the timer - this advanced tool can estimate your performance and suggest more practice questions. We have subscribed you to Daily Prep Questions via email.

Customized
for You

we will pick new questions that match your level based on your Timer History

Track
Your Progress

every week, we’ll send you an estimated GMAT score based on your performance

Practice
Pays

we will pick new questions that match your level based on your Timer History
Not interested in getting valuable practice questions and articles delivered to your email? No problem, unsubscribe here.
Close
Request Expert Reply
Confirm Cancel
SORT BY:
Date
Math Expert
Joined: 02 Sep 2009
Posts: 92920
Own Kudos [?]: 619085 [26]
Given Kudos: 81596
Send PM
Most Helpful Reply
Math Expert
Joined: 02 Sep 2009
Posts: 92920
Own Kudos [?]: 619085 [5]
Given Kudos: 81596
Send PM
General Discussion
Manager
Manager
Joined: 12 Mar 2017
Posts: 185
Own Kudos [?]: 88 [0]
Given Kudos: 87
Location: India
Concentration: Strategy, General Management
GMAT 1: 700 Q49 V37
GPA: 4
Send PM
Target Test Prep Representative
Joined: 24 Nov 2014
Status:Chief Curriculum and Content Architect
Affiliations: Target Test Prep
Posts: 3480
Own Kudos [?]: 5137 [1]
Given Kudos: 1431
GMAT 1: 800 Q51 V51
Send PM
Re: V21-08 [#permalink]
1
Kudos
Expert Reply
Prateek176 wrote:
I did not understand what is wrong with option E. The explanation above doesn't seem clear enough

I tend to agree with what the explanation says about (E).

To complete the passage, we need a reason why the economists sided with the policymakers.

Let's look at (E):

E. most policymakers of that time considered federal regulation of the market to be an emergency measure that, as such, should only be temporary

(E) gives us no information regarding why the economists would side with the policymakers. It tells us only why most policymakers were against regulation of the market.

By the way, another issue with (E) is that the argument is about reasons for discontinuing the existence of certain agencies, while (E) is about regulating the market, which seems to be a topic different from discontinuing the existence of agencies.
Manager
Manager
Joined: 26 Aug 2017
Posts: 50
Own Kudos [?]: 107 [2]
Given Kudos: 696
Location: Singapore
GMAT 1: 710 Q49 V37
GMAT 2: 760 Q50 V44
WE:General Management (Health Care)
Send PM
Re: V21-08 [#permalink]
2
Kudos
My two cents' worth here, happy to hear any feedback or discussion on this:

We need to explain the economists' point of view and need to have reasons potentially different from that of the policymakers but when combined with other opinions, the case for terminating the agencies is justified.

Reasoning is:

1) If the economists agreed with the federal govt policymakers, and that they feared economic depression but actually still agreed to get rid of the agencies, the reason could be:

- Not related to the fear which the policymakers have
- Somehow related to the possibility that if the agencies stayed longer, it would do more harm than if it was discontinued

2) Note that the stem says "feared another economic depression" meaning the depression is not here yet, AND this fear may not necessarily be linked to the existence of the agencies i.e. it could just be a general fear ANY economist would have in general.

3) Therefore, the reason must be something else e.g. say the agencies are not fit for purpose anymore NOW, compared to the time the agencies were first created (to administer financial relief) --> Option B says this.

Error analysis:

- A is irrelevant and contradictory to the direction we need.
- C could be true, but is irrelevant again to why the agencies should be terminated.
- For D, it is unclear if having a well-expanded power and authority of the agencies is DETRIMENTAL OR BENEFICIAL - we only know that the agencies' powers have expanded, but we're not sure if that effect should get the agencies to stay or go. Not MATERIAL impact.
- E does not explain the economists' point of view.

Answer = B.
Math Expert
Joined: 02 Sep 2009
Posts: 92920
Own Kudos [?]: 619085 [0]
Given Kudos: 81596
Send PM
Re: V21-08 [#permalink]
Expert Reply
The question or/and solution has been revised and edited. Thank you sayantanc2k !!!
User avatar
Non-Human User
Joined: 09 Sep 2013
Posts: 32679
Own Kudos [?]: 822 [0]
Given Kudos: 0
Send PM
Re: V21-08 [#permalink]
Hello from the GMAT Club BumpBot!

Thanks to another GMAT Club member, I have just discovered this valuable topic, yet it had no discussion for over a year. I am now bumping it up - doing my job. I think you may find it valuable (esp those replies with Kudos).

Want to see all other topics I dig out? Follow me (click follow button on profile). You will receive a summary of all topics I bump in your profile area as well as via email.
GMAT Club Bot
Re: V21-08 [#permalink]
Moderator:
Math Expert
92918 posts

Powered by phpBB © phpBB Group | Emoji artwork provided by EmojiOne